Please help - Binary Relations driving me utterly insane

  • Thread starter Thread starter Mathmos6
  • Start date Start date
  • Tags Tags
    Binary Relations
Click For Summary
The discussion revolves around proving the equivalence of four conditions related to the composition of two equivalence relations, R and S, on a set A. The user has made progress by showing that condition (i) implies (ii) and that (i) is equivalent to (iii), but struggles with proving that (ii) implies (i). They also indicate difficulty in establishing the relationship between the conditions and the uniqueness of the smallest equivalence relation containing both R and S. Ultimately, the user expresses frustration but concludes that they have resolved their main concern.
Mathmos6
Messages
76
Reaction score
0

Homework Statement



If R and S are two equivalence relations on the same set A, we define R ◦ S =
{(x, z ) ∈ A × A : there exists y ∈ A such that (x, y) ∈ R and (y, z ) ∈ S }.

Show that the following conditions are equivalent:
(i) R ◦ S is a symmetric relation on A ;
(ii) R ◦ S is a transitive relation on A ;
(iii) S ◦ R ⊆ R ◦ S ;
(iv) R ◦ S is the unique smallest equivalence relation on A containing both R and S .

The Attempt at a Solution



I've spent literally hours trying to solve this and my brain is leaking out my ears now :( I've managed to prove (i -think-) that (i)=>(ii), and that (i)<=>(iii), but I can't see any way whatsoever to show (ii)=>(i), and I've managed to show that R ◦ S contains both R and S for (iv) but I don't know how to show that if it's the smallest such relation (iv)<=>(i) or (ii) or (iii).

Please help! I'm quite a way out of my depth - (ii)=>(i) is the most frustrating bit, because I'm sure it's probably really obvious but I just can't seem to get it out! :(
 
Last edited:
Physics news on Phys.org
Never mind, got it.
 
Question: A clock's minute hand has length 4 and its hour hand has length 3. What is the distance between the tips at the moment when it is increasing most rapidly?(Putnam Exam Question) Answer: Making assumption that both the hands moves at constant angular velocities, the answer is ## \sqrt{7} .## But don't you think this assumption is somewhat doubtful and wrong?

Similar threads

  • · Replies 3 ·
Replies
3
Views
1K
  • · Replies 6 ·
Replies
6
Views
2K
  • · Replies 1 ·
Replies
1
Views
2K
  • · Replies 3 ·
Replies
3
Views
2K
  • · Replies 1 ·
Replies
1
Views
2K
  • · Replies 3 ·
Replies
3
Views
2K
  • · Replies 1 ·
Replies
1
Views
2K
  • · Replies 2 ·
Replies
2
Views
3K
  • · Replies 8 ·
Replies
8
Views
3K
Replies
1
Views
2K